Examine the convergence \(\Large\int\limits_{0}^{2}\frac{dx}{2x^{2}-x^{2}}\)


A) converges

B) diverges

C) converges to

D) none of these

Correct Answer:
B) diverges

Description for Correct answer:
\(\Large \frac{1}{2x-x^{2}}=\frac{1}{x(2-x)}\)

0, 2 are the points of infinite discontituity.

Now \(\Large \int\limits_{0}^{2}\frac{dx}{2x-x^{2}}\)

\(\Large =\lim\limits_{\lambda\rightarrow 0+}\int\limits_{\lambda}^{1}\frac{dx}{x(2-x)}+\lim\limits_{\mu\rightarrow 0+}\int\limits_{1}^{2-\mu}\frac{2x}{2(2-x)}\)

\(\Large \frac{1}{2}\lim\limits_{\lambda\rightarrow 0+}\left[ log\frac{x}{2-x} \right]_{\lambda}^{1}+\frac{1}{2}\lim\limits_{\mu\rightarrow 0+}\left[ log\frac{x}{2-x} \right]_{\lambda}^{2-\mu}\)

\(\Large =-\frac{1}{2}\lim\limits_{\lambda\rightarrow 0+}\frac{\lambda}{2-\lambda}+\frac{1}{2}\lim\limits_{\mu\rightarrow 0+}\frac{2-\mu}{\mu}\)

\(\Large =\infty\)

\(\therefore\) The given integral diverges.

Part of solved Real Analysis questions and answers : >> Elementary Mathematics >> Real Analysis








Comments

No comments available




Similar Questions
1). The integral \(\Large\int\limits_{1}^{\infty}\frac{dx}{x(x+1)}\)
A). log 2
B). log 3
C). log 5
D). log 7
-- View Answer
2). Examine the convergence of \(\Large\int\limits_{0}^{\pi}\frac{dx}{1+cosx}\).
A). convergent
B). converges to 1
C). converges to 0
D). diverges
-- View Answer
3). Let \(f(x)=x,\ 0\le x\le 1\) and \(p=0,\frac{1}{4},\frac{1}{2},\frac{3}{4},1\) be the partition of [0, 1]. Find U(p, f) and L(p, f).
A). \(\Large \frac{1}{8},\frac{1}{8}\)
B). \(\Large \frac{3}{8},\frac{7}{8}\)
C). \(\Large \frac{5}{8},\frac{3}{8}\)
D). \(\Large \frac{7}{8},\frac{9}{8}\)
-- View Answer
4). Let f(x) be defined on [0, 1] as follows:\[ f(x) = 1\begin{cases}\text{1 when } x \text{ is rational}\\\text{-1 when } x\text{ is irrational}\end{cases}\]
A). Riemann integrable
B). not Rimann integrable
C). continuous
D). none of these
-- View Answer
5). Let \(f(x)=\frac{1}{x}\ 0\le x\le 2\ P=\{ 1,1.2,1.4,1.6,1.8,2 \}\) Find \(w(P,f)\)
A). 0,1
B). 0.2
C). 0.3
D). none of these
-- View Answer


6). \(f(x)=x^{2}\). Find \(\int\limits_{\overline{0}}^{a}\) and \(\int\limits_{n}^{\overline{0}}f\)
A). \(\Large \frac{a}{3},\frac{a}{3}\)
B). \(\Large \frac{a^{2}}{3},\frac{a^{2}}{3}\)
C). \(\Large \frac{a^{3}}{3},\frac{a^{3}}{3}\)
D). none of these
-- View Answer
7). Any countable infinite Set is equivalent to a
A). subset
B). proper subset
C). null set
D). none of these
-- View Answer
8). \(Q \times Q\) is
A). uncountable
B). countable
C). finitgset
D). infinite set
-- View Answer
9). (0, 1] is
A). countable
B). uncountable
C). finite
D). none of these
-- View Answer
10). The set of irrational numbers lying in the interval (0, 1] is
A). countable
B). finite
C). uncountable
D). none of these
-- View Answer